'Triangular Distributions' Probability Density Function

Click For Summary
The discussion centers on triangular distributions, specifically their probability density functions (PDFs) defined by piece-wise linear functions. The PDF is characterized by parameters a, b, and c, where f(a) and f(b) equal zero, and f(c) represents the maximum value. Participants express confusion over deriving numerical answers without specific values for a, b, and c, emphasizing that results will remain symbolic. The relationships between the constants K and L are crucial for defining the PDF, as they ensure the integral of the function equals one. Overall, understanding the triangular distribution requires grasping its structure and the relationships between its defining parameters.
pone
Messages
6
Reaction score
0
(\Triangular" distributions.) Let X be a continuous random variable with prob-
ability density function f(x). Suppose that all we know about f is that a </= X </= b,
f(a) = f(b) = 0, and that there exists a value c between a and b where f is at a maxi-
mum. A natural density function to consider in this case is a piece-wise linear function,
corresponding to lines connecting (a; 0) with (c; f(c)), and (c; f(c)) with (b; 0).
a) What is the value of f(c)?
b) Sketch a graph of f(x).
c) Compute the expected value E(X) and the variance Var(X).

I have not been given any numbers and am very confused as to how there could be a numerical answer to this question. I know the probability density function looks like a triangle, with f(a) and f(b) on the x-axis, but am not sure where to go with this. Anyone have a suggestion?
 
Physics news on Phys.org
The density function will look like:

f(x)=K(x-a) a<x<c
f(x)=L(b-x) c<x<b

where L and K are determined by:
L(b-c)=K(c-a)
integral from a to b of f(x)=1.

Your results will be functions of a, b and c, so don't expect to get numbers unless a, b, and c are specified.
 
Last edited:
Question: A clock's minute hand has length 4 and its hour hand has length 3. What is the distance between the tips at the moment when it is increasing most rapidly?(Putnam Exam Question) Answer: Making assumption that both the hands moves at constant angular velocities, the answer is ## \sqrt{7} .## But don't you think this assumption is somewhat doubtful and wrong?

Similar threads

  • · Replies 19 ·
Replies
19
Views
4K
  • · Replies 11 ·
Replies
11
Views
2K
Replies
6
Views
2K
Replies
2
Views
2K
  • · Replies 2 ·
Replies
2
Views
2K
  • · Replies 4 ·
Replies
4
Views
2K
Replies
1
Views
2K
  • · Replies 1 ·
Replies
1
Views
2K
Replies
2
Views
1K
  • · Replies 5 ·
Replies
5
Views
2K